A question on exact sequences [duplicate]












0












$begingroup$



This question already has an answer here:




  • Characterization of short exact sequences

    2 answers




Let $M^{prime}xrightarrow{f}Mxrightarrow{g}M^{primeprime}rightarrow 0 DeclareMathOperator{Hom}{Hom}$ be a sequence of $A$-modules and homomorphisms. I want to show that the sequence is exact if and only if for all $A$-modules $N$, the sequence $$0rightarrow Hom(M^{primeprime},N)xrightarrow{overline{g}} Hom(M,N)xrightarrow{overline{f}} Hom(M^{prime},N)$$ is exact.



I have shown that $M^{prime}xrightarrow{f}Mxrightarrow{g}M^{primeprime}rightarrow 0$ is exact implies $0rightarrow Hom(M^{primeprime},N)xrightarrow{overline{g}} Hom(M,N)xrightarrow{overline{f}} Hom(M^{prime},N)$ is exact for all $A$-modules $N$.



Now suppose for all $A$-modules $N$, $0rightarrow Hom(M^{primeprime},N)xrightarrow{overline{g}} Hom(M,N)xrightarrow{overline{f}} Hom(M^{prime},N)$ is exact. I want to show that $M^{prime}xrightarrow{f}Mxrightarrow{g}M^{primeprime}rightarrow 0$ is exact. That is I have to show $g$ is onto and $ker(g)=operatorname{Image}(f)$. In Atiyah Macdonald's Commutative Algebra, it is written that since $overline{g}$ is injective for all $A$-modules $N$, therefore $g$ is onto. But I could not understand it. Any explanation will be appreciated.










share|cite|improve this question











$endgroup$



marked as duplicate by Paul Frost, A. Pongrácz, KReiser, Martín Vacas Vignolo, Takumi Murayama Dec 31 '18 at 2:06


This question has been asked before and already has an answer. If those answers do not fully address your question, please ask a new question.














  • 3




    $begingroup$
    This has been asked many times. Please search!
    $endgroup$
    – user26857
    Dec 30 '18 at 16:40










  • $begingroup$
    See for example math.stackexchange.com/q/375763 and math.stackexchange.com/q/1301692.
    $endgroup$
    – Paul Frost
    Dec 30 '18 at 17:05
















0












$begingroup$



This question already has an answer here:




  • Characterization of short exact sequences

    2 answers




Let $M^{prime}xrightarrow{f}Mxrightarrow{g}M^{primeprime}rightarrow 0 DeclareMathOperator{Hom}{Hom}$ be a sequence of $A$-modules and homomorphisms. I want to show that the sequence is exact if and only if for all $A$-modules $N$, the sequence $$0rightarrow Hom(M^{primeprime},N)xrightarrow{overline{g}} Hom(M,N)xrightarrow{overline{f}} Hom(M^{prime},N)$$ is exact.



I have shown that $M^{prime}xrightarrow{f}Mxrightarrow{g}M^{primeprime}rightarrow 0$ is exact implies $0rightarrow Hom(M^{primeprime},N)xrightarrow{overline{g}} Hom(M,N)xrightarrow{overline{f}} Hom(M^{prime},N)$ is exact for all $A$-modules $N$.



Now suppose for all $A$-modules $N$, $0rightarrow Hom(M^{primeprime},N)xrightarrow{overline{g}} Hom(M,N)xrightarrow{overline{f}} Hom(M^{prime},N)$ is exact. I want to show that $M^{prime}xrightarrow{f}Mxrightarrow{g}M^{primeprime}rightarrow 0$ is exact. That is I have to show $g$ is onto and $ker(g)=operatorname{Image}(f)$. In Atiyah Macdonald's Commutative Algebra, it is written that since $overline{g}$ is injective for all $A$-modules $N$, therefore $g$ is onto. But I could not understand it. Any explanation will be appreciated.










share|cite|improve this question











$endgroup$



marked as duplicate by Paul Frost, A. Pongrácz, KReiser, Martín Vacas Vignolo, Takumi Murayama Dec 31 '18 at 2:06


This question has been asked before and already has an answer. If those answers do not fully address your question, please ask a new question.














  • 3




    $begingroup$
    This has been asked many times. Please search!
    $endgroup$
    – user26857
    Dec 30 '18 at 16:40










  • $begingroup$
    See for example math.stackexchange.com/q/375763 and math.stackexchange.com/q/1301692.
    $endgroup$
    – Paul Frost
    Dec 30 '18 at 17:05














0












0








0





$begingroup$



This question already has an answer here:




  • Characterization of short exact sequences

    2 answers




Let $M^{prime}xrightarrow{f}Mxrightarrow{g}M^{primeprime}rightarrow 0 DeclareMathOperator{Hom}{Hom}$ be a sequence of $A$-modules and homomorphisms. I want to show that the sequence is exact if and only if for all $A$-modules $N$, the sequence $$0rightarrow Hom(M^{primeprime},N)xrightarrow{overline{g}} Hom(M,N)xrightarrow{overline{f}} Hom(M^{prime},N)$$ is exact.



I have shown that $M^{prime}xrightarrow{f}Mxrightarrow{g}M^{primeprime}rightarrow 0$ is exact implies $0rightarrow Hom(M^{primeprime},N)xrightarrow{overline{g}} Hom(M,N)xrightarrow{overline{f}} Hom(M^{prime},N)$ is exact for all $A$-modules $N$.



Now suppose for all $A$-modules $N$, $0rightarrow Hom(M^{primeprime},N)xrightarrow{overline{g}} Hom(M,N)xrightarrow{overline{f}} Hom(M^{prime},N)$ is exact. I want to show that $M^{prime}xrightarrow{f}Mxrightarrow{g}M^{primeprime}rightarrow 0$ is exact. That is I have to show $g$ is onto and $ker(g)=operatorname{Image}(f)$. In Atiyah Macdonald's Commutative Algebra, it is written that since $overline{g}$ is injective for all $A$-modules $N$, therefore $g$ is onto. But I could not understand it. Any explanation will be appreciated.










share|cite|improve this question











$endgroup$





This question already has an answer here:




  • Characterization of short exact sequences

    2 answers




Let $M^{prime}xrightarrow{f}Mxrightarrow{g}M^{primeprime}rightarrow 0 DeclareMathOperator{Hom}{Hom}$ be a sequence of $A$-modules and homomorphisms. I want to show that the sequence is exact if and only if for all $A$-modules $N$, the sequence $$0rightarrow Hom(M^{primeprime},N)xrightarrow{overline{g}} Hom(M,N)xrightarrow{overline{f}} Hom(M^{prime},N)$$ is exact.



I have shown that $M^{prime}xrightarrow{f}Mxrightarrow{g}M^{primeprime}rightarrow 0$ is exact implies $0rightarrow Hom(M^{primeprime},N)xrightarrow{overline{g}} Hom(M,N)xrightarrow{overline{f}} Hom(M^{prime},N)$ is exact for all $A$-modules $N$.



Now suppose for all $A$-modules $N$, $0rightarrow Hom(M^{primeprime},N)xrightarrow{overline{g}} Hom(M,N)xrightarrow{overline{f}} Hom(M^{prime},N)$ is exact. I want to show that $M^{prime}xrightarrow{f}Mxrightarrow{g}M^{primeprime}rightarrow 0$ is exact. That is I have to show $g$ is onto and $ker(g)=operatorname{Image}(f)$. In Atiyah Macdonald's Commutative Algebra, it is written that since $overline{g}$ is injective for all $A$-modules $N$, therefore $g$ is onto. But I could not understand it. Any explanation will be appreciated.





This question already has an answer here:




  • Characterization of short exact sequences

    2 answers








commutative-algebra modules exact-sequence






share|cite|improve this question















share|cite|improve this question













share|cite|improve this question




share|cite|improve this question








edited Dec 30 '18 at 16:38









Bernard

124k742117




124k742117










asked Dec 30 '18 at 16:23









AnupamAnupam

2,5741925




2,5741925




marked as duplicate by Paul Frost, A. Pongrácz, KReiser, Martín Vacas Vignolo, Takumi Murayama Dec 31 '18 at 2:06


This question has been asked before and already has an answer. If those answers do not fully address your question, please ask a new question.









marked as duplicate by Paul Frost, A. Pongrácz, KReiser, Martín Vacas Vignolo, Takumi Murayama Dec 31 '18 at 2:06


This question has been asked before and already has an answer. If those answers do not fully address your question, please ask a new question.










  • 3




    $begingroup$
    This has been asked many times. Please search!
    $endgroup$
    – user26857
    Dec 30 '18 at 16:40










  • $begingroup$
    See for example math.stackexchange.com/q/375763 and math.stackexchange.com/q/1301692.
    $endgroup$
    – Paul Frost
    Dec 30 '18 at 17:05














  • 3




    $begingroup$
    This has been asked many times. Please search!
    $endgroup$
    – user26857
    Dec 30 '18 at 16:40










  • $begingroup$
    See for example math.stackexchange.com/q/375763 and math.stackexchange.com/q/1301692.
    $endgroup$
    – Paul Frost
    Dec 30 '18 at 17:05








3




3




$begingroup$
This has been asked many times. Please search!
$endgroup$
– user26857
Dec 30 '18 at 16:40




$begingroup$
This has been asked many times. Please search!
$endgroup$
– user26857
Dec 30 '18 at 16:40












$begingroup$
See for example math.stackexchange.com/q/375763 and math.stackexchange.com/q/1301692.
$endgroup$
– Paul Frost
Dec 30 '18 at 17:05




$begingroup$
See for example math.stackexchange.com/q/375763 and math.stackexchange.com/q/1301692.
$endgroup$
– Paul Frost
Dec 30 '18 at 17:05










1 Answer
1






active

oldest

votes


















1












$begingroup$

Hint:



Take for $N$ the cokernel of $g$, i.e. consider the (surjective) canonical map
$$M''longrightarrow M''/g(M)$$
and deduce from the hypothesis that $M''=g(M)$.






share|cite|improve this answer











$endgroup$




















    1 Answer
    1






    active

    oldest

    votes








    1 Answer
    1






    active

    oldest

    votes









    active

    oldest

    votes






    active

    oldest

    votes









    1












    $begingroup$

    Hint:



    Take for $N$ the cokernel of $g$, i.e. consider the (surjective) canonical map
    $$M''longrightarrow M''/g(M)$$
    and deduce from the hypothesis that $M''=g(M)$.






    share|cite|improve this answer











    $endgroup$


















      1












      $begingroup$

      Hint:



      Take for $N$ the cokernel of $g$, i.e. consider the (surjective) canonical map
      $$M''longrightarrow M''/g(M)$$
      and deduce from the hypothesis that $M''=g(M)$.






      share|cite|improve this answer











      $endgroup$
















        1












        1








        1





        $begingroup$

        Hint:



        Take for $N$ the cokernel of $g$, i.e. consider the (surjective) canonical map
        $$M''longrightarrow M''/g(M)$$
        and deduce from the hypothesis that $M''=g(M)$.






        share|cite|improve this answer











        $endgroup$



        Hint:



        Take for $N$ the cokernel of $g$, i.e. consider the (surjective) canonical map
        $$M''longrightarrow M''/g(M)$$
        and deduce from the hypothesis that $M''=g(M)$.







        share|cite|improve this answer














        share|cite|improve this answer



        share|cite|improve this answer








        edited Jan 1 at 15:41









        user26857

        39.6k124284




        39.6k124284










        answered Dec 30 '18 at 16:47









        BernardBernard

        124k742117




        124k742117















            Popular posts from this blog

            Biblatex bibliography style without URLs when DOI exists (in Overleaf with Zotero bibliography)

            ComboBox Display Member on multiple fields

            Is it possible to collect Nectar points via Trainline?